Finding an approximate formula for f'(a)

Click For Summary
The discussion revolves around finding an approximate formula for the derivative f'(a) using Taylor Series Expansion. The initial approach involved expanding f(a+h) and f(a+3h) and forming a linear combination of f(a), f(a+h), and f(a+3h). Feedback indicated that the derived expression for f'(a) was incorrect, prompting a suggestion to simplify the Taylor expansions to second order to eliminate f''(a). Ultimately, the user confirmed they arrived at the correct answer and inquired about identifying the error term in their calculations. The conversation highlights the importance of careful calculation and understanding error terms in Taylor series approximations.
theBEAST
Messages
361
Reaction score
0

Homework Statement


fPv6h.png


The Attempt at a Solution


q6sC7.jpg


Alright so what I did was expanded f(a+h) and f(a+3h) using Taylor Series Expansion. I then said that f'(a) would be some linear combination of f(a), f(a+h) and f(a+3h).

I summed up and factored out the terms f(a), f'(a) and f''(a) from the Taylor Series Expansions.

Finally for equations 1, I know that I don't want f(a) so c0+c1+c2 = 0. For equation 2, I want to keep f'(a) so that equation is set to equal to 1. For equation 3, I don't want f''(a) so that equation is set to 0.

I solved for the coefficients in terms of h and ended up with f'(a) ≈ [f(a+3h) - 33f(a) + 27f(a+h)] / 20hI would like to know if my method and/or answer is correct. My answer in my opinion looks really weird.
 
Physics news on Phys.org
You have a good idea, you're just calculating it wrong. The expression for f'(a) is not right. Do it again more carefully.

Another, slightly easier approach would be to write the Taylor expansions for f(a+h) and f(a+3h) up to second order and then using these two, eliminate f''(a) and solve for f'(a).
 
clamtrox said:
You have a good idea, you're just calculating it wrong. The expression for f'(a) is not right. Do it again more carefully.

Another, slightly easier approach would be to write the Taylor expansions for f(a+h) and f(a+3h) up to second order and then using these two, eliminate f''(a) and solve for f'(a).

Thanks! I got the correct answer. Out of curiosity, which term would be the error term? the sum of the f''(a) terms or the sum of the f'''(a) terms?
 
Question: A clock's minute hand has length 4 and its hour hand has length 3. What is the distance between the tips at the moment when it is increasing most rapidly?(Putnam Exam Question) Answer: Making assumption that both the hands moves at constant angular velocities, the answer is ## \sqrt{7} .## But don't you think this assumption is somewhat doubtful and wrong?

Similar threads

  • · Replies 2 ·
Replies
2
Views
2K
  • · Replies 9 ·
Replies
9
Views
3K
  • · Replies 9 ·
Replies
9
Views
2K
  • · Replies 2 ·
Replies
2
Views
2K
  • · Replies 8 ·
Replies
8
Views
2K
  • · Replies 3 ·
Replies
3
Views
1K
  • · Replies 3 ·
Replies
3
Views
2K
  • · Replies 8 ·
Replies
8
Views
2K
Replies
2
Views
2K
  • · Replies 3 ·
Replies
3
Views
2K